56
J2 H2 Economics (9732/01) 1 PAPER 1 Friday 16 September 2016 08:00 – 10:15 TIME : 2 hours 15 mins INSTRUCTIONS TO CANDIDATES Do not open this paper until you are told to do so. Write your name, class and name of economics tutor in the space provided on the writing paper. Do not use staples, paper clips, glue or correction fluid/tape. Answer all questions. The number of marks is given in the brackets at the end of each question. Write your answers on the writing papers provided. If you use more than one sheet of paper, fasten the sheets together. Submit the answers for both case study questions separately. You are advised to spend several minutes per question reading through the data and questions before you begin writing your answers. There are _8_ printed pages including this cover page PRELIMINARY EXAM 2016 Economics JC2 H2 (9732/01) NYJC

2016 Economics JC2 H2 (9732/01) NYJC

  • Upload
    others

  • View
    5

  • Download
    1

Embed Size (px)

Citation preview

Page 1: 2016 Economics JC2 H2 (9732/01) NYJC

J2 H2 Economics (9732/01) 1

PAPER 1

Fr iday 16 September 2016 08:00 – 10:15 TIME : 2 hours 15 mins

INSTRUCTIONS TO CANDIDATES

Do not open this paper until you are told to do so.

Write your name, class and name of economics tutor in the space provided on the writing paper.

Do not use staples, paper clips, glue or correction fluid/tape.

Answer all questions. The number of marks is given in the brackets at the end of each question. Write your answers on the writing papers provided. If you use more than one sheet of paper, fasten the sheets together. Submit the answers for both case study questions separately.

You are advised to spend several minutes per question reading through the data and questions before you begin writing your answers.

There are _8_ printed pages including this cover page

PRELIMINARY EXAM 2016

Economics JC2 H2 (9732/01)

NYJC

Page 2: 2016 Economics JC2 H2 (9732/01) NYJC

J2 H2 Economics (9732/01) 2

Question 1: A Sweet Affair Sugar is one of the oldest traded commodities in the world. It can be refined from sugar cane or sugar beet. Sugar from cane accounts for eighty percent of all sugar produced in the world whereas the remaining twenty percent comes from sugar beet. Sugar from beet and cane is seen as the same, consequently there is no difference in sugar prices between the two. Extract 1: Sugar Surges Most in Twenty-Two Years Sugar prices surged the most in at least twenty-two years after the International Sugar Organization increased its forecast for a production deficit in the current crop year amid rising concern about the impact of the El Nino weather pattern on supplies. El Nino has already hurt plantings in Brazil the largest sugar grower, as well as India and Thailand, spurring price gains in the final months of 2015. It’s not just adverse weather that’s supporting prices. Brazilian mills are making more ethanol from sugar cane to meet surging domestic demand for the biofuel. In addition, the expansion of food processing industry in China, India and Russia is also adding extra pressure on sugar prices.

Source: Adapted from Bloomberg, 23 February 2016

Figure 1: World Sugar Price (US Cents per Pound)

Source: IndexMundi, accessed 26 July 2016 Extract 2: Soft Drinks, Hard Questions The British government announced in March that it intends to introduce a tax on sugary drinks in 2018 to tackle childhood obesity. The United Kingdom is not alone in worrying about the global rise in obesity-related ailments such as diabetes and cardiovascular disease — costly conditions that can lead to significant disability and early death. Many countries have introduced, or are considering, taxes on unhealthy foods and drinks — in particular, those to which sugar has been added. These food taxes operate one of the most effective behavioural-change levers available to policymakers: Price. But it would be premature to conclude that higher prices necessarily lead to lower consumption and thus better health outcomes. A tax’s effectiveness depends on how it is designed, and how consumers and the food industry respond to the incentives it creates.

Page 3: 2016 Economics JC2 H2 (9732/01) NYJC

J2 H2 Economics (9732/01) 3

The introduction of taxes on sugary drinks in Mexico and Berkeley, California, has lowered profits and the industry is resisting the legislation. Evidence suggests that these taxes, once implemented, did indeed result in a change in price for customers, who subsequently bought fewer sugary drinks. Unfortunately, however, little is known about the effect of these taxes on public health. Obesity and its related ailments take a long time to develop, and isolating the effects of food taxes from changes caused by other health policies and cultural trends is challenging. The debate has already begun. The UK soft drink industry is considering legal action, arguing that the tax is anti-competitive given that pure fruit juice and sweetened milk are not included.

Source: Adapted from Today Online, 28 July 2016

Extract 3: Soft Drinks Groups Digest UK Sugar Tax The outlook is less sweet for soft drinks companies after a long-dreaded sugar tax was revealed on Wednesday by George Osborne, the UK chancellor. Mr Osborne said he expected the levy to raise £520m in its first year and slightly less than £1.5bn over three years. That is a sizeable amount for an industry whose revenues — which last year were £16bn — have fallen in two out of the past three years. The share prices of the UK’s biggest drinks manufacturers fell sharply on Wednesday and saw slides on Thursday as analysts said they expected further sales declines as a result of the new tax. Coca-Cola Enterprises, the Coke bottler in the UK, has the biggest share of the UK market, thirty-six percent of its carbonate sales. However, the UK is less than two percent of the Coca-Cola Company’s overall business. Britvic is the second-largest producer, with sixteen percent of the soft drinks market. It acts as bottler for PepsiCo and has its own drinks. Charles Pick, analyst at Numis, says: “If the levy is passed on to the consumer, as it probably will be, most brands will see a short-term fall in consumption.” Ian Shackleton at Nomura, calculates that raising £520m implies a high twenty-five to thirty percent increase to retail selling prices.

Source: Adapted from Financial Times, 17 March 2016

Figure 2: UK Carbonated Drinks Market

Source: Adapted from Euromonitor, 20 March 2016

Page 4: 2016 Economics JC2 H2 (9732/01) NYJC

J2 H2 Economics (9732/01) 4

Extract 4: Sugar Market Braced for EU’s Return Many sugar producing countries protect their domestic industries through some combination of guaranteed payments to farmers, production restrictions or import limits. But the industry is gradually liberalizing in a series of moves that could push the price of sugar lower. That has refiners and exporters scrambling to work out what newly opened markets will mean for them. In the biggest change, the European Union will remove production and export quotas for its sugar-beet farmers from October 2017. Together with other measures, the quotas were meant to promote greater self-sufficiency of sugar in the EU market and price stabilisation that ensured the profitability of sugar-beet farmers. Sugar producers in Europe, currently the world’s third largest, say they plan to produce more as a result of the removal of the quotas. The EU may then turn from a net importer to an exporter. More than two-thirds of European sugar is produced from its sugar beet, with the remaining from imported sugar cane refined in EU and imports from other sources. The change will benefit beet growers who are cost-efficient and can scale up their production at competitive prices. How much sugar the EU will be able to sell overseas, however, will depend on the international price compared with the production costs. Claudiu Covrig, an analyst at sugar consultants Kingsman, says the EU sugar groups will be returning to an already competitive international market. For sugar cane farmers, especially those in the bloc of African, Caribbean and Pacific (ACP) countries that has historically benefitted from the EU’s managed market, the future looks grim. Many ACP countries seem likely to be squeezed out of the market as they are considered to be uncompetitive vis-à-vis sugar beet producers. As high-cost producers with few alternative markets, they will need to undergo radical restructuring in order to preserve industry revenue and jobs, or else transition out of sugar cane altogether.

Source: Various, 2016

Answer all questions (a) (i) Describe the trend in world sugar prices between June 2015 and June 2016.

[2]

(ii) With the help of diagrams, explain how the surging demand for biofuel may have resulted in the trend described in (a)(i).

[4]

(b) (i) Describe the type of market structure operating in the UK carbonated drinks market.

[2]

(ii) Explain how firms in the UK carbonated drinks market will compete. [4]

(c) Assess the possible impacts of the removal of “production and export quotas for its sugar beet farmers” mentioned in Extract 4 on the EU and ACP countries.

[8]

(d) Extract 2 mentioned about the increasing worry on the global rise in obesity-related ailments. Discuss if the tax on sugary drinks will necessarily lead to “a better health outcome”.

[10]

[Total: 30 marks]

Page 5: 2016 Economics JC2 H2 (9732/01) NYJC

J2 H2 Economics (9732/01) 5

Question 2: Concerns over China’s Slowdown Extract 5: EU Sees Weaker Growth in Eurozone and Wider EU as China Slowdown Weighs

Figure 3: GDP changes in Europe

Source: European Commission Growth in the Eurozone and the wider European Union will be slightly weaker this year than previously forecast, the European Commission predicted Tuesday, warning that the economic slowdown in China and other emerging markets, geopolitical tensions and uncertainty ahead of the U.K. referendum on EU membership could weigh on the economy. The EU’s economists also cautioned that the impact of factors that have been supporting growth in the region, such as low oil prices and a weaker euro, could start to fade. Fundamental problems in many of the bloc’s economies, including high levels of private debt and unemployment, continue to hold back the recovery, they said. The EU economy expanded 1.7% in 2015.

Source: Adapted from Wall Street Journal, 3 May 2016

Extract 6: How Exposed Is Europe To A Chinese Economic Slowdown? China has become one of the EU’s key external trading partners in goods. It ranks second overall in terms of total trade and in particular has been a key source of demand for exports in recent years. At a time when the Eurozone is struggling, it should not be under-estimated as a source of external demand and cheap inputs. While it is important in terms of goods trade, total trade in services is relatively limited. It is clear from the above that Europe does have some exposure to China. Any slowdown in China and emerging markets more broadly might hamper the fragile economic recovery which is in place in the Eurozone. Additionally, while China is one of the EU’s largest external trade

Page 6: 2016 Economics JC2 H2 (9732/01) NYJC

J2 H2 Economics (9732/01) 6

partners, it ranks fairly low for most individual countries in terms of trade as much of their trade is with other EU states or the US. Germany has brushed off concerns that turmoil in China could impact its economy with a spokeswoman for the Economy Ministry saying the “immediate consequences” should be “limited”. One of the reasons behind this could be that net exports have actually been a less important driver of German economic growth than domestic consumption since 2012. Germany has also shown impressive export flexibility in the face of crisis – when Eurozone demand for its exports halted it was able to shift towards China and other markets; it could yet try to pull off such a move again. For example, it has shown some success in tapping into the nascent US recovery with exports growing quickly in past few months. Similarly, the surprisingly impressive recovery in Spain has been driven by consumption and domestic demand rather than net exports, let alone demand from Asian markets.

Source: Adapted from Forbes, 25 August 2015

Extract 7: China's Slowdown Could Be a Plus for U.S. and Europe in the End China’s slowdown is blamed for causing everything from global market turmoil to falling sales of crocodile-skin handbags. Yet the slowest growth in 25 years in the world’s second-biggest economy is proving a boost for consumers and companies in Western Europe and the U.S., according to Neville Hill, an economic analyst in London. When China grew at double-digit rates, its voracious demand for materials drove up commodity and energy prices. That hurt the buying power of consumers in Western economies and weighed on corporate sentiment as rising costs hurt profits. Now, that situation is being reversed. Plunging commodity prices are boosting European and American shoppers and spurring corporate earnings growth. "China is exporting deflation, but for the West it is good deflation rather than bad deflation in that the cost of the stuff we buy has gone down" Hill said.

Source: Adapted from Bloomberg, 4 November 2015

Extract 8: China's Slowdown will Hit Singapore Hardest: ANZ Warns The slowdown in China will hit Singapore faster and harder than any country in the region, economists from Australian bank ANZ have warned. Other economists were not as dire in their prognoses, but agreed that the open nature of the local economy and the strong trade, investment and tourism links between the two nations mean that slower growth on the mainland will have a direct and broad-based impact on businesses and jobs here. "Singapore's economy has been on a structural shift - getting more dependent on China - over the last 10 to 20 years," said DBS economist Irvin Seah. Unlike its Asean neighbours, Singapore does not have a big domestic market to act as a buffer against weaker demand from China, said Mr Seah. But DBS and HSBC take a slightly more positive view. "Singapore's manufacturing sector is already in recession, but domestic services are holding up relatively well," said HSBC economist Joseph Incalcaterra. He noted that services exports, even those to China, are still growing.

Page 7: 2016 Economics JC2 H2 (9732/01) NYJC

J2 H2 Economics (9732/01) 7

Other than slower growth, Chinese import demand is also sliding, partly because of the weak yuan, said Barclays economist Leong Wai Ho. This could have a larger-than-expected impact on China's trading partners and suppliers in Asia, which will hit Singapore factories and worsen the ongoing manufacturing recession, he added.

Source: Adapted from Straits Times, 12 Jan 2016

Figure 4: Singapore Exports To Major Partners(S$bn)

0

1

2

3

4

5

6

7

2015 Jul 2015 Aug 2015 Sep 2015 Oct 2015 Nov 2015 Dec 2016 Jan

China Europe Malaysia USA

Source: Department of Statistics, Singapore Extract 9: Singapore Must Prepare for Economic Slowdown: PM Lee With the global economy facing cyclical headwinds, Prime Minister Lee Hsien Loong has warned that Singapore must brace itself to handle a possible downturn. Advances in technology, meanwhile, is also disrupting industries and displacing workers at all levels, be they blue-collar workers in the factories or professionals such as lawyers and doctors. "We know our direction, (which is) to improve productivity so that we can sustain higher wages for all. But we need to review specific measures - how to help our domestic sectors grow, how to attract investments and help companies develop new markets, and how to make best use of the foreign workers and talent that we need in Singapore," he said. Overall, Singapore is in a better position than most other countries to tackle the tough challenges ahead. Mr Lee highlighted the Republic's strengths such as a well-educated population, an ethos that is outward-looking, a tech-savvy society and competent unions.

Source: Adapted from Business Times, 27 October 2015

Page 8: 2016 Economics JC2 H2 (9732/01) NYJC

J2 H2 Economics (9732/01) 8

Table 1: Singapore: Selected Indicators

Indicators (Quarter-on-Quarter) 2015 1Q

2015 2Q

2015 3Q

2015 4Q

2016 1Q

Change in GDP At 2010 Market Prices (%) 0.2 -1.6 2.3 6.2 0.2 Workers made redundant in Manufacturing 950 870 920 2,480 1,790 Workers made redundant in Services 1,930 2,100 2,120 2,360 2,530 Net Exports Of Goods And Services (bn) 29.4 26.9 25.6 29.9 25.7 CPI (Base Year 2014 = 100) 99.9 99.7 99.4 99.2 98.9 Tourism Receipts (S$m) 5,314 5,039 6,039 5,385 na

Source: Department of Statistics, Singapore

Answer all questions (a) (i) Compare the economic growth of the countries shown in Fig 3 from 2012 to

2015. [2]

(ii) Account for the economic growth trend of Eurozone over the same period. [3]

(b) (i) Explain why economists consider deflation to be undesirable. [3]

(ii) With the aid of a diagram, explain why the London analyst used the term “good deflation” in Extract 7.

[4]

(c) In view of the current economic conditions, Monetary Authority of Singapore announced a policy change to zero appreciation of currency in January 2016. Comment on the appropriateness of this policy for the year 2016.

[8]

(d) With reference to the data where appropriate, discuss the view that China’s economic slowdown will impact Singapore more than Europe.

[10]

[ Total 30 marks ]

********** The End **********

Page 9: 2016 Economics JC2 H2 (9732/01) NYJC

(a) (i) Describe the trend in world sugar prices between June 2015 and June 2016.

Overall increasing trend (1) Sharpest increase between April 2016 to June 2016 (1) or any other

anomaly

[2]

(ii) With the help of diagrams, explain how the surging demand for biofuel may have resulted in the trend described in (a)(i).

Sugar cane can be used to produce either biofuel or sugar (Extract 1, para2), biofuel and sugar are in competitive supply. (1)

Surging demand for biofuel Demand for biofuel ↑ Eqm price and qtyof biofuel ↑ + Diagram (1)

Increased eqm qty of biofuel requires more sugar cane Demand forsugar cane ↑ Price of sugar cane ↑ (1)

Price of sugar cane ↑ cost of producing sugar from sugar cane ↑ Supply ↓ Price of sugar ↑ (Diagram) Leading to rising trend (1)

[4]

(b) (i) Describe the type of market structure operating in the UK carbonated drinks market.

Oligopoly (1) From Figure 2, five-firm concentration ratio is 84%. (1)

[2]

(ii) Explain how firms in the UK carbonated drinks market will compete.

Interdependence Firms will take into account actions of rival firms (1) Explain how total revenue will fall due to price competition (1)

o Price ↓ Rival firms will also ↓ price Inelastic portion of thekinked demand curve Qd ↑ by less than proportionate TR ↓

Thus, engage in non-price competition to increase demand and/or makedemand more price inelastic. (1)

Non-price competition strategy (1)o Advertising to create imaginary differences & develop brand loyaltyo R&D to create real differences, new products, higher quality

products etc.

[4]

(c) Assess the possible impacts of the removal of “production and export quotas for its sugar beet farmers” mentioned in Extract 4 on the EU and ACP countries.

Impact on market for sugar beet With a quota of Qo, eqm price and qty of sugar beet were Po and Qo

[8]

Page 10: 2016 Economics JC2 H2 (9732/01) NYJC

With the removal of quotas, supply of sugar beet can be determined by market

forces, eqm price of sugar beet ↓ from Po to Pe and eqm qty ↑ from Qo to Qe. Impact on market for sugar With a lower eqm price on sugar beet, COP for producing sugar from sugar

beet ↓ Supply of sugar in EU ↑

Macro impacts on EU With higher production of sugar, EU may turn from a net importer to exporter

(Extract 4, para 2).

Before the quota was lifted, domestic price of sugar PDom was above the world

price Pw. EU was a net importer and imported Q2-Q1 amount of sugar. With the removal of quota on sugar beet, supply of sugar by EU ↑ from SDom to

SDom’, new domestic price PDom’ is lower than Pw. EU becomes a net exporter and exports Q3-Q2 amount to the world market.

However, if supply of sugar by EU only ↑ from SDom to SDom’’, the new domestic price PDom’’ is still higher than Pw. EU remains as a net importer but imports a smaller amount of Q2-Q4.

With a ↑ in export revenue / ↓ in import expenditure, AD ↑ RNY ↑, unemployment ↓

Sugar from sugar beet and sugar from sugar cane are perfect substitute. As supply of sugar from sugar beet ↑ and could be sold at a lower price, sugar from sugar cane loses its competitiveness in the EU market. Demand for sugar from sugar cane ↓

About one third of EU’s sugar is produced from imported sugar cane and imports from other sources (Extract 4, para 2). Although unemployment in EU falls, sugar cane importers and refiners in the EU may go out of business and be unemployed.

Page 11: 2016 Economics JC2 H2 (9732/01) NYJC

Macro impacts on ACP countries Exports of sugar cane by ACP countries to EU ↓ as sugar cane is more

expensive than sugar beet (Extract 4, para 4). In addition, as price of sugar beet ↓ due to higher production by EU, coupled

with the lifting of export quota on sugar beet, sugar producers in other parts of the world may also substitute sugar cane with the cheaper sugar beet ↑ competition from sugar beet by EU Exports of sugar cane to other countries also ↓

Export revenue of ACP countries ↓ AD ↓ RNY ↓ Sugar cane farmers become unemployed.

Evaluation Actual impacts on EU and ACP countries depend on the magnitude of the ↓ in

price of sugar beet after the quotas are removed. Impacts on EU are largely positive as RNY and employment ↑, although sugar

cane exporters and refiners in EU are likely to be the losers of the policy. Impacts on the ACP countries are clearly negative as they are the high cost

producers who are uncompetitive. Extent of impact depends on actions taken by the govt.

Level Knowledge, Application, Understanding & Analysis Marks L2 A well-developed answer that analyses the micro impacts on

the sugar beet and sugar markets and the macro impacts on both EU and ACP, supported with usage of analytical tools and evidence from extracts. An answer that only analyses the macro impacts on EU and ACP.

4-6 Max 4

L1 An underdeveloped answer that focuses on the micro impacts without sufficient explanation on the macro impacts. Or One sided answer that analyses only the impacts on EU or ACP countries.

1-3

E Application of relevant economic concepts to make judgment on the overall impacts to EU and ACP countries.

1-2

(d) Extract 2 mentioned about the increasing worry on the global rise in obesity-related ailments. Discuss if the tax on sugary drinks will necessarily lead to “a better health outcome”. “A better heath outcome” refers to a fall in the incidence of obesity-related

ailments such as diabetes and cardiovascular disease Explain that sugary drink is a demerit good. Sugary drinks deemed to be socially undesirable by the govt (Extract 2, para

1) demerit good

[10]

Page 12: 2016 Economics JC2 H2 (9732/01) NYJC

Explain the negative externality in consumption of sugary drink and its relation to obesity-related ailments Explain the negative externality in consumption of sugary drinks

Explain the MPB, MPC and MEC in the context of sugary drink (Extract 2, para

1) Explain that given MEC, MSC is greater than MPC Qp is greater than Qs

Overconsumption and DWL Market has failed Overconsumption of sugary drinks ↑ obesity-related ailments. Explain the imperfect information in the consumption of sugary drink and its relation to obesity related ailments Explain the MPC and MPB in the context of sugary drink

Explain the imperfect info and the divergence between MPCPerceived and MPCActual.

Explain that MPCPerceived is higher than MPCActual, QP is greater than QA Overconsumption and DWL Market has failed

Overconsumption of sugary drinks ↑ obesity-related ailments. Thesis: An indirect tax will lead to better health outcome An indirect tax can be imposed to correct the negative externality in

consumption.

MPC

MPB 

Qp 

Cost/Benefit 

MEC 

MSC

= MSB 

QS 

Qty of sugary drinks consumed

Over-consumption 

MPCPerceived 

MPB 

QPerceived 

Cost/Benefit  MPCActual

QActual 

Qty of sugary drinks consumed

Page 13: 2016 Economics JC2 H2 (9732/01) NYJC

When a tax equal to MEC is imposed, MPC will rise by the MEC amount and coincide with MSC New Qp, Qp’ coincides with Qs Consumption at Qs, DWL is eliminated.

Consumption of sugary drink now at the socially optimal level Leads to better health outcome.

Provide evidence from Ext 4 to illustrate how tax has resulted in higher prices and consumers subsequently bough fewer sugary drinks.

Anti-Thesis: An indirect tax will not lead to better health outcome Government may undertax due to

o Government failure govt doesn’t know the extent of MEC o Demand for sugary drinks could be price inelastic amount of tax

required is high Due to undertax, overconsumption of sugary drink is not solved (Explain with

diagram) Will not lead to better health outcome. An indirect tax could not solve the root cause for imperfect information

Measures to provide information on the true cost of consuming sugary drinks, such as education, health campaign etc is needed to solve the imperfect information.

With the measures, MPCPerceived coincides with MPCActual New QPerceived coincides with QActual Consumption at QActual DWL is eliminated.

Consumption of sugary drink now at the optimal level Leads to better health outcome.

Evaluation: Ceteris paribus assumptions may not hold, other reasons also affect health outcome Reasons other than consumption of sugary drink will also affect health outcome

(Extract 2, para 3) Other health policies e.g. healthy lifestyle campaigns etc may also affect health outcome Hard to isolate the effect of tax.

Other sugary drinks such as fruit juice and sweetened milk are not subjected to

MPC 

MPB 

Qp 

Cost/Benefit 

MEC 

MSC = MPC + tax  

Qs = Qp’ 

Qty of sugary drinks consumed

MPCPerceived 

MPB 

QPerceived 

Cost/Benefit  MPCActual = MPCPerceived’

QActual = QPerceived’ 

Qty of sugary drinks consumed  

Page 14: 2016 Economics JC2 H2 (9732/01) NYJC

the sugar tax (Extract 2, para 4) consumers may substitute sugary soft drink with these other drinks consumption of sugar remains unchanged May not lead to better health outcome �

Level Knowledge, Application, Understanding & Analysis Marks L3 A well-developed, balanced answer on the effects of a tax to

tackle both sources of market failure with explicit link to health outcome. Well supported with usage of analytical tools and evidence.

7-8

L2 An underdeveloped analysis on the effects of a tax to both sources of market failure, without sufficient references to case materials. Or A balanced analysis that only analyses the effects of a tax in solving negative externality in consumption or imperfect information Or One-sided answer that analyses only the effectiveness or limitation of a tax An answer that analyses the effects of a tax without explaining the source of market failure.

4-6 Max 4

L1 For a descriptive answer that makes little reference to economic theory, or makes reference to economic theory but with little application to the context. Answer is likely to be one-sided.

1-3

E Application of relevant economic concepts to make judgment about the effectiveness of a tax and recognises that there are other factors affecting the health outcome.

1-2

[Total: 30 marks]

Page 15: 2016 Economics JC2 H2 (9732/01) NYJC

Question 2  Concerns Over China’s Slowdown 

Extract 5: EU Sees Weaker Growth in Eurozone and Wider EU as China Slowdown Weighs 

Growth in the Eurozone and the wider European Union will be slightly weaker this year than previously forecast, the European Commission predicted Tuesday, warning that the economic slowdown in China and other emerging markets, geopolitical tensions and uncertainty ahead of the U.K. referendum on EU membership could weigh on the economy. The EU’s economists also cautioned that the impact of factors that have been supporting growth in the region, such as low oil prices and a weaker euro, could start to fade. Fundamental problems in many of the bloc’s economies, including high levels of private debt and unemployment, continue to hold back the recovery, they said. The EU economy expanded 1.7% in 2015. 

Wall Street Journal May 3, 2016 (Adapted) 

Extract 6:  How Exposed Is Europe To A Chinese Economic Slowdown? 

China has become one of the EU’s key external trading partners in goods. It ranks second overall in terms of total trade and in particular has been a key source of demand for exports in recent years. At a time when the Eurozone is struggling, it should not be under‐estimated as a source of external demand and cheap inputs. While it is important in terms of goods trade, total trade in services is relatively limited. 

It is clear from the above that Europe does have some exposure to China. Any slowdown in China and emerging markets more broadly might hamper the fragile economic recovery which is in place in the Eurozone. Additionally, while China is one of the EU’s largest external trade partners, it ranks fairly low for most individual countries in terms of trade as much of their trade is with other EU states or the US. 

Germany has brushed off concerns that turmoil in China could impact its economy with a spokeswoman for the Economy Ministry saying the “immediate consequences” should be “limited”. One of the reasons behind this could be that net exports have actually been a less important driver of German economic growth than domestic consumption since 2012. Germany has also shown impressive export flexibility in the face of crisis – when Eurozone demand for its exports halted it was able to shift towards China and other markets; it could yet try to pull off such a move again. For example, it has shown some success in tapping into the nascent US recovery with exports growing quickly in past few months. Similarly, the surprisingly impressive recovery in Spain has been driven 

Page 16: 2016 Economics JC2 H2 (9732/01) NYJC

by consumption and domestic demand rather than net exports, let alone demand from Asian markets. 

FORBES   Aug 25, 2015 (Adapted)    

 

Extract 7:  China's Slowdown Could Be a Plus for U.S. and Europe in the End 

China’s slowdown is blamed for causing everything from global market turmoil to falling sales of crocodile‐skin handbags. Yet the slowest growth in 25 years in the world’s second‐biggest economy is proving a boost for consumers and companies in Western Europe and the U.S., according to Neville Hill, an economic analyst in London. 

When China grew at double‐digit rates, its voracious demand for materials drove up commodity and energy prices. That hurt the buying power of consumers in Western economies and weighed on corporate sentiment as rising costs hurt profits. 

Now, that situation is being reversed. Plunging commodity prices are boosting European and American shoppers and spurring corporate earnings growth. "China is exporting deflation, but for the West it is good deflation rather than bad deflation in that the cost of the stuff we buy has gone down, but the price we receive for our labour has gone up," Hill said. 

BLOOMBERG  4 Nov 2015 (Adapted)  

Extract 8:  China's slowdown will hit Singapore hardest: ANZ warns 

The slowdown in China will hit Singapore faster and harder than any country in the region, economists from Australian bank ANZ have warned. 

Other economists were not as dire in their prognoses, but agreed that the open nature of the local economy and the strong trade, investment and tourism links between the two nations mean that slower growth on the mainland will have a direct and broad‐based impact on businesses and jobs here. "Singapore's economy has been on a structural shift ‐ getting more dependent on China ‐ over the last 10 to 20 years," said DBS economist Irvin Seah. Unlike its Asean neighbours, Singapore does not have a big domestic market to act as a buffer against weaker demand from China, said Mr Seah.  

But DBS and HSBC take a slightly more positive view. "Singapore's manufacturing sector is already in recession, but domestic services are holding up relatively well," said HSBC economist Joseph Incalcaterra. He noted that services exports, even those to China, are still growing. 

Other than slower growth, Chinese import demand is also sliding, partly because of the weak yuan, said Barclays economist Leong Wai Ho. This could have a larger‐than‐expected impact on China's trading partners and suppliers in Asia, which will hit Singapore factories and worsen the ongoing manufacturing recession, he added. 

Straits Times  Jan 12, 2016 (Adapted) 

Page 17: 2016 Economics JC2 H2 (9732/01) NYJC

 

 

Extract 9: Singapore must prepare for economic slowdown: PM Lee 

WITH the global economy facing cyclical headwinds, Prime Minister Lee Hsien Loong has warned that Singapore must brace itself to handle a possible downturn. 

Advances in technology, meanwhile, is also disrupting industries and displacing workers at all levels, be they blue‐collar workers in the factories or professionals such as lawyers and doctors.  

"We know our direction, (which is) to improve productivity so that we can sustain higher wages for all. But we need to review specific measures ‐ how to help our domestic sectors grow, how to attract investments and help companies develop new markets, and how to make best use of the foreign workers and talent that we need in Singapore," he said. 

Overall, Singapore is in a better position than most other countries to tackle the tough challenges ahead. Mr Lee highlighted the Republic's strengths such as a well‐educated population, an ethos that is outward‐looking, a tech‐savvy society and competent unions. 

Business Times OCT 27, 2015 (Adapted) 

 

 

Table 1            Singapore: Selected Indicators

Indicators (Quarter‐on‐Quarter) 2015 1Q 

2015 2Q 

2015 3Q 

2015 4Q 

2016 1Q 

Change in GDP At 2010 Market Prices (%)  0.2 ‐1.6 2.3 6.2  0.2 

Workers made redundant in  Manufacturing   950 870 920 2,480  1,790 

Workers made redundant in  Services  1,930 2,100 2,120 2,360  2,530 

Net Exports Of Goods And Services (bn)  29.4 26.9 25.6 29.9  25.7 

CPI (Base Year 2014 = 100) 99.9 99.7 99.4 99.2  98.9 

Tourism Receipts (S$m) 5,314 5,039 6,039 5,385 na 

Department of Statistics Singapore 

 

 

 

0

1

2

3

4

5

6

7

2015 Jul 2015 Aug 2015 Sep 2015 Oct 2015 Nov 2015 Dec 2016 Jan

Fig 3    Singapore Exports To Major Partners(S$bn)

China Europe Malaysia USA

Page 18: 2016 Economics JC2 H2 (9732/01) NYJC

Questions  

a   i  Compare the economic growth of the countries shown in Fig 3 from 2012 to 2015.  2 

ii  Account for the economic growth trend of Eurozone over the same period.  3

b  i  Explain why economists consider deflation to be undesirable  3 

ii  With the aid of a diagram, explain why the London analyst used the term “good deflation” in Extract 7. 

c  In view of the current economic conditions, Monetary Authority of Singapore announced a policy change to zero appreciation of currency in January 2016. 

Comment on the appropriateness of this policy for the year 2016. 

d  With reference to the data where appropriate, discuss the view that China’s economic slowdown will impact Singapore more than Europe. 

10 

  Total  30

Suggested Answers 

a  i  Compare the economic growth of the countries shown in Fig 3 from 2012 to 2015.  2

  Increasing trend for all countries (1 mark) 

Germany’s economic growth was always positive compared to other countries  

OR 

only Greece is forecasted to have negative growth in 2015 compared to other countries (any one reason 1 mark) 

 

a ii  Account for the economic growth trend of Eurozone over the same period.  3

  Eurozone growth is generally increasing for the same period. (1mark ) 

This can be due to low oil prices or weak Euro currency . 

Explain how low oil price lead to growth  (2 marks ) 

OR 

Explain how weak Euro contributes to growth (2 marks )  

(diagram not needed ) 

 

b  i  Explain why economists consider deflation to be undesirable.  3 

  Define deflation – sustained/persistent decrease in GPL (1 mark) 

If the consumers expect that, the fall in price will continue they will tend to delay purchases as they wait for lower, more attractive prices This in turn causes falling output and investment  producers may start retrenching workers leading 

 

Page 19: 2016 Economics JC2 H2 (9732/01) NYJC

to rising unemployment.

During inflation borrower’s gain but during deflation they lose as the value of debt is higher. 

Easier for employers to control real wages when there is positive inflation however, it is difficult to cut wages when there is deflation. 

(well elaborated any one reason (2 marks ) 

b ii  With the aid of a diagram, explain why the London analyst used the term “good deflation” in Extract 7. 

  Evidence (Extract 7 last para) 

 Lower EG  in China caused China to reduce demand for energy and commodities. This reduces the world price of energy  and commodities and hence reduces cost of production for other countries. (2 marks ) (Cannot accept  lower COP for other countries  because of raw materials imported from China  has become cheaper  – the effect must be shown via reduced global prices of energy and commodities  ) 

Illustrate using AD/AS model: 

AD/AS Diagram (1 mark ) 

AS increase  SRAS shifts downwards GPL fall yet RNY rises 

Good deflation because although prices are falling there is increase in RNY .(1mark)  

 

c  In view of the current economic conditions, Monetary Authority of Singapore announced a policy change to zero appreciation of currency in January 2016. 

Comment on the appropriateness of this policy for the year 2016. 

  Identify the possible problems faced by Spore in 2016 ( can be from case study or outside) 

Table 1: Singapore: Selected Indicators

Indicators (Quarter-on-Quarter) 2015 1Q 2015 2Q 2015

3Q 2015 4Q 2016 1Q

Change in GDP At 2010 Market Prices (%) 0.2 -1.6 2.3 6.2 0.2

Workers made redundant in Manufacturing 950 870 920 2,480 1,790

Workers made redundant in Services 1,930 2,100 2,120 2,360 2,530

Net Exports Of Goods And Services (bn) 29.4 26.9 25.6 29.9 25.7

CPI (Base Year 2014 = 100) 99.9 99.7 99.4 99.2 98.9

Tourism Receipts (S$m) 5,314 5,039 6,039 5,385 na

From Table 1 identify the problems Singapore is currently facing: low economic growth, falling tourism receipts, falling exports, increasing unemployment. 

The policy of  zero appreciation of currency can help address  this  fall. Singapore 

Page 20: 2016 Economics JC2 H2 (9732/01) NYJC

Government  generally  uses  a  policy  of modest  appreciation  to maintain  price stability. However, with CPI falling in 2016 imported inflation is less of a concern, and slow growth is a more pressing issue.  

Zero appreciation means that MAS has taken the Singapore dollar off the path of modest 

and gradual appreciation and has completely flattened the slope of the band. 

Thus the policy of zero appreciation coupled with deflation will make our exports more  competitive  and  can  also  make  investments  less  expensive  in  terms  of foreign currency. This will also encourage tourism as tourists need to spend less in terms of their own currency  increase X and I component of AD and hence cause RNY to increase ceteris paribus leading to economic growth.  

This will also address the unemployment problem arising in export sector. 

Anti‐thesis  

Structural unemployment (Ext 8) will not be addressed. 

Zero appreciation of currency may not be sufficient to address slowdown in major export markets, as X is about 200% of GDP.  

Other countries may have depreciated their currencies also therefore making Singapore relatively less competitive.  

The lower export prices may not be sufficient to counter the fall in demand from trading partners because of fall in income. 

Evaluate :Suggest an alternative policy that can address the problems faced in 2016 (policies that addresses other components of AD e.g. I & C.) 

Refer to extract 5 ‐ Need to “help our domestic sectors  grow, attract investments and help companies develop new markets” 

Help domestic firms to expand – Fiscal policies like tax allowances for automation, grants for investing in technology, etc 

Attract Investments ‐ LR Supply side policy to upgrade infrastructure, FP to reduce tax burden 

Develop new markets – Trade policy with other countries beside China – Europe seems a promising mkt according to Fig 2.:  FTAs:  

Raise outward FDI to raise returns from foreign investment to reduce reliance on exports. 

Structural Une – training of PMETs, Career conversion programmes.  

  

L2  Two sided answer that looks at the effectiveness of zero appreciation of currency in the light of problems mentioned in the data. 

4‐6 

L1  One sided discussion that explains the appropriateness of zero appreciation of currency  Or superficial  two sided explanation without case study references 

1‐3 

E  One alternative policy suggestion that address the problems identified above 

1‐2 

      

Page 21: 2016 Economics JC2 H2 (9732/01) NYJC

 

d  With reference to the data where appropriate, discuss the view that China’s economic slowdown will impact Singapore more than Europe. 

10

  Intro : Explain how China’s economic slowdown will impact Singapore and Europe in terms of macro‐economic variables like economic growth and employment.  Thesis :China’s economic slowdown will impact Singapore more than Europe because  (Need to justify why the impact on Singapore is higher as well as explain why Europe may not be affected significantly ) 

Spore has been getting more dependent on China in terms of trade, investment and tourism. (Evidence from Extract 4). Moreover Singapore doesn’t have a  big domestic market to buffer against fall in external demand (China being a major trading partner ) 

On the other hand the impact of China’s economic slowdown on Europe may not be very significant because of the following reasons  

Europe conducts more trade  with US and within Europe 

Their main source of EG comes from domestic economy rather than exports eg. Spain,  

Trade in services is limited  

Evidence was seen in  Germany’s ability to adjust – shift to other markets like USA 

Good deflation – rising real incomes and profits in Europe  

Anti thesis: China’s economic slowdown might impact Europe more than Singapore 

because :  (Need to justify why the impact on Singapore may not be so significant  as well as explain  why Europe may be affected significantly )  

Europe’s exposure to China is quite high – key trading partner. 

Germany accounts for half of EU exports to China 

 

Page 22: 2016 Economics JC2 H2 (9732/01) NYJC

China provides cheap inputs  ‐ affect cost of production  

The forecast for the Eurozone countries is higher growth in figure 1. 

Impact on Singapore may not be significant because  

Service sector is resilient.  

Service export to China is still growing. 

Good deflation will benefit Spore too 

Spore in a better position due to its well educated workforce and high tech population. 

 Judgement In the short run, Spore may be in a better position to absorb the impact due to its ability to adjust quickly and low unemployment. Europe is saddled with high unemployment and high debt that makes adjustment difficult. However, in the long run, Europe, with her large domestic market will be able to offset China’s slowdown while Spore will still be reliant on Exports.  

L3  Well‐developed and balanced answer that clearly discusses how China’s economic slowdown will impact Singapore and Europe. Makes good reference to data. 

7‐8 

L2  Underdeveloped answer that considers both Singapore and Europe but with gaps in explanation and analysis.  

Or Answer that discusses the impact of China’s slowdown on either Singapore or Europe (two sided views) 

OR  One‐sided answer for both countries with reference to data. 

4‐6 

L1  A sketchy answer that refers to some data to explain how China’s slowdown affects either countries. 

1‐3 

E  A well‐developed evaluative judgment on the validity of the argument. 

1‐2 

 

 

Page 23: 2016 Economics JC2 H2 (9732/01) NYJC

J2 H2 Economics (9732/02) 1

PRELIMINARY EXAM 2016

JC2 Economics

H2 (9732/02) NYJC

Paper 2 - Essay Tuesday 20 September 2016 08:00 – 10:15 TIME: 2 hours 15 mins INSTRUCTIONS TO CANDIDATES Write your name, class and name of economics tutor in the space provided on the writing paper. Do not use staples, paper clips, glue or correction fluid/tape. Answer three questions in total, of which one must be from Section A, one from Section B and one from either Section A or Section B. The number of marks is given in the brackets at the end of each question or part question. Write your answers on the writing papers provided. At the end of the examination, fasten all your work securely together.

There are __2_ printed pages including this cover page

Page 24: 2016 Economics JC2 H2 (9732/01) NYJC

J2 H2 Economics (9732/02) 2

Answer three questions in total.

Section A

One or two of your three chosen questions must be from this section.

1 Many countries are reporting economic growth due to an increase in merchandise trade, technology transfer and labour mobility.

Assess the impact of these developments on different product markets in Singapore. [25]

2 (a) Explain how firms can increase market power. [10] (b) Discuss the extent to which the performance of a firm is determined by its market

power. [15]

3 Shisha smoking was banned in Singapore in 2014 because smokers were at risk of

developing the same health problems as cigarette smokers, such as cancer and heart disease.

(a)

Explain why government intervention is advocated in the markets for both merit and demerit goods.

[10]

(b) Discuss whether a ban is the best response in addressing the market failure arising

from shisha smoking. [15]

Section B One or two of your three chosen questions must be from this section. 4 Amid the slowest GDP growth in six years and the flattest employment growth in more

than a decade, Finance Minister Heng Swee Keat announced a Budget with a strong focus on economic restructuring such as tightening the foreign labour market to raise productivity.

Source: channelnewsasia.com/news/singapore/budget-2016

(a)

Explain the key determinants of sustained economic growth.

[10]

(b) To what extent should the Singapore government focus on raising productivity to

achieve sustained economic growth? [15]

5 (a) Explain the different types of unemployment an economy can face. [10] (b) Discuss the view that structural unemployment is the most significant

macroeconomic problem faced by open economies. [15]

6 Globalisation has made the planet more equal. As communication gets cheaper and transport gets faster, developing countries have closed the gap with their rich-world counterparts. But within many developing economies, inequality has worsened. Discuss the effects of globalisation on the standard of living in the different economies.

[25]

Page 25: 2016 Economics JC2 H2 (9732/01) NYJC

Demand & Supply

1 "Many countries are reporting growth due to an increase in world merchandise trade, technology transfer and labour mobility.”

Assess the impact of these recent developments on different product markets in Singapore.

[25]

Suggested Approach

Interpretation of Question What is the command cue? (What are the skills required for the question?)

“Assess” – a 2-sided analysis, and evaluation of likely outcomes in the different markets with justification.

What are the content cues? (What are the concepts required to answer the question)

“recent developments”: - “increase in world merchandise trade” economic growth increase in income change in demand depending on the nature of good

- “technological transfer” lower cost of production increase supply of goods - “labour mobility” reduce pressure on wages lower cost of production increase supply of goods

“impact of … on markets”: - impact on equilibrium price, quantity, total revenue

What are the contextual cues? (What is the context for the question?)

“Different product markets inSingapore” – luxury, necessity and inferior products sold in Singapore

Introduction Issue The experience by many countries based on the preamble are essentially traits of

globalization. Growth in world merchandise trade result in economic growth rise in

income for households. Increase in trade increase in exports and imports availability of wider range

of consumer goods and services. Increase in transfer of technology and labour mobility increase productivity and

reduce wage pressures lowering cost of production.

Approach This essay aims to discuss the possible impact of the above developments on different product markets in Singapore, using demand and supply analysis.

The relevant economic concepts to include for analysis are: PED, YED, PES and shifts in DD and SS curves.

Body

The product markets here refer to the luxury, necessities and inferior goods markets in Singapore.

Page 26: 2016 Economics JC2 H2 (9732/01) NYJC

The rise in income arising from the increase in world merchandise trade and capital flows shifts the demand curve to the right for luxury goods and necessities while the demand for inferior good falls, shifting the demand curve leftward.

The supply curve for all product markets increase due to the lower cost of production arising from higher productivity from technology transfer and the influx of labour. More goods are also available from increased imports due to increase in trading activities and this increases the supply of goods as well.

Elaboration

Product Market

∆ in Demand when Y↑(Consider YED & PES)

∆ in Supply (Consider PED)

Final impact on the product market (Evaluate which outcome is most likely)

Luxury goods (e.g. designer clothes or watches) (Justify YED>1)

Demand↑ more than proportionately

PES < 1 assuming luxury goods are of top quality and take longer production period + relatively smaller number of producers (niche markets) (PES > 1 also possible) DD↑ P↑, Q↑

PED > 1 due to the nature of the good + higher proportion of income

SS↑ P↓, Q↑ (more than prop.)

DD↑ > SS↑: P↑, Q↑, TR↑

Necessities (e.g. food) (Justify 0<YED<1)

Demand↑ less than proportionately

PES < 1 assuming necessities such as fresh food which have shorter shelf lives and difficult to accumulate stocks

DD↑ P↑, Q↑ Ev: P↑ but to a smaller extent compared to luxury goods 

PED<1 as they are basic goods + smaller proportion to income

SS↑ P↓, Q↑ (less than prop.)

DD↑ < SS↑: P↓, Q↑, TR↑

Inferior goods (e.g. 2nd hand cars or non-smart phones) (Justify YED<0)

Demand↓ with a rise in income.

PES > 1 assuming they are low-end manufactured goods with stocks that are easily accumulated

DD↓ P↓, Q↓

PED>1 assuming many substitutes

SS↑ P↓, Q↑ (more than prop.)

DD↓ + SS↑ P↓, Q?, TR? Effect on Q and TR depends on the magnitude of shifts of dd and ss (e.g. Q can ↓ if dd for inferior goods is income elastic, leading to TR↓)

Page 27: 2016 Economics JC2 H2 (9732/01) NYJC

Evaluation

Question ceteris paribus assumption – In reality, ceteris paribus assumption does not hold and many factors may change concurrently to influence the impact on the product markets.

Difficulty of getting accurate elasticity data

Conclusion

The impact on the different product markets (luxury, necessities, inferior) differ when there is an increase in world merchandise trade, capital flows, technology transfer and labour mobility, depending on the relative shifts of demand and supply. In the case of luxury goods, it is clear that both price and quantity will increase but not necessarily so for necessities though the case for inferior good is likely to see a fall in both price and quantity.

Despite the limitations of using elasticity concepts, the above analysis can still be useful as a guide for businesses in Singapore in deciding the industries to focus on, the choice of goods to produce and if there is a need to restructure or divest in view of the developments in order to remain competitive.

Note: • Students are free to justify the PED/PES of the goods concerned as long as they are conceptually sound and reasonable. And that will have impact on the final outcome of the product markets.

Page 28: 2016 Economics JC2 H2 (9732/01) NYJC

Mark Scheme

Level Descriptors Level 3 18 – 21 15 – 17

• Answer shows excellent knowledge of demand and supply forces and how these affect the 3 different product markets namely luxury, necessity and inferior products. • Clear analysis of simultaneous shifts, recognition that equilibrium price or quantity may be reinforced and in another instance, it may be indeterminate. • Answer is balanced in consideration of demand and supply forces. Answer includes the different extent of shifts of supply vs demand. • Excellent rigour in economic analysis and development. • Answer shows good knowledge of demand and supply forces and how these affect the 3 different product markets. • Good consideration of elasticity concepts (i.e. YED and PED &/or PES) and its relevance in influencing market equilibrium. • Good use of diagrams that is adequately explained (which includes diagrams with simultaneous shifts). Max 15m if answer shows knowledge of the 3 markets, but only does combined shift for 1.

Level 2 12 – 14 10 – 11

• Answer shows adequate knowledge of demand and supply forces and how these affect equilibrium price and quantity. • Some consideration of elasticity concepts (i.e. at least one elasticity concept well explained) and its relevance in influencing market equilibrium. • Some rigour in economic analysis and development. • Max of 13m if only 2 out of 3 markets are well analysed. • Answer is relevant but analysis has some inaccuracies/incomplete examples. Is able to identify forces in DD and SS but lacking in detailed elasticity discussions. • But answer lacks balance in consideration of demand and supply forces.

Level 1 6 – 9 1 – 5

• Able to sketch a DD & SS diagram and identify & briefly explain impact on the product markets. • Some errors and inconsistencies occur in the explanation and use of some economic concepts. • Minimal reference to context. • Misinterpreted question as a macroeconomic issue but there are few valid points though they do not clearly address the question. • Demonstrates some basic knowledge of different types of products.

E2 (3 – 4)

Explained judgement with economic analysis with regards to the impact on different product markets resulting from the combined effects of change in demand and supply. Include the ability to question assumptions.

E1 (1 – 2)

Mainly unexplained judgement.

Page 29: 2016 Economics JC2 H2 (9732/01) NYJC

MARKET STRUCTURE 

(a) Explain how firms can increase market power?   [10] (b) Discuss the extent to which the performance of firms is determined by their market 

power.    [15] 

Approach: Students are expected to explain various strategies that firms may employ to increase 

their market power. Answers can categorised under demand and supply factors (At least 3) 

Suggested outline 

Introduction Market power refers to a firm’s ability to influence the market price without losing all of its sales.  Market power is measured by the no of producers, size of the firms, level of barriers to entry and the availability of substitute goods. Firms can increase their market power through erecting artificial barriers to entry through strategic entry deterrence and control of raw materials. Firms can also seek to increase their market power by mergers with other firms. 

 Body Demand factors Mergers  

Firms can also increase their market power through mergers with other firms. A mergerrefers to two or more companies or organisations combining together, resulting in a larger firm.  

Duplication of operations is removed to reduce costs. Cost advantage over rivals, enablesfirm to increase its market power.  

Merged firm controls a larger market share, better able to raise prices.

Eg.

Strategic deterrence 

Incumbent firms engage in aggressive advertising campaigns to increase brand loyalty andbrand awareness amongst its consumers.  

Advertising campaigns ‐ leads to higher demand for product, increasing the consumer basefor firm.  

Make the demand for the firm’s product relatively price‐inelastic due to greater brandloyalty.  

Deters potential competitors from entering industry, as new firms would have to incur asizeable cost for advertising (BTE) to be able to successfully penetrate the market. 

Pursuing other objectives like output‐maximising to raise customer base and increasemarket share, or enable firm to enjoy IEOS when producing a large output (especially for new firms). Includes predatory pricing when firm is trying to drive out competition in a foreign market. Aim is to reap long‐run profits by sacrificing short‐run profits. 

Eg.

Page 30: 2016 Economics JC2 H2 (9732/01) NYJC

Supply Factors Control of raw materials 

Firms can also increase their market power by controlling necessary raw materials.  

Will limit any entry of potential firms or existing rivals could face rising/higher cost of production. This will increase the market power of the incumbent firm. 

Eg. DeBeers    

Conclusion  

  

Level  Descriptors Marks

  L1  

For  a  descriptive  answer with  no  or  limited  link  to  how  firms  can  increase market power.  Concept errors.  

 1‐4 

   L2  

For an underdeveloped (lacking in scope or depth) answer that explains at least 2 methods firms can use to increase their market power.   Mere description of barriers with no  linkage  to what  firms  can do  to protect these advantages – max 5 marks     

 5‐6 

 L3  

For a well‐developed answer that explains at least three methods firms can use to increase their market power. With examples provided  3 well‐developed points with brief examples – max 8 marks   Well‐developed explanation with at  least 3  factors and no examples – max 7 marks   

 7‐10 

                  

Page 31: 2016 Economics JC2 H2 (9732/01) NYJC

(b) Discuss the extent to which the performance of firms is determined by their market power.                                                          [15]  

 Approach: Students are expected to highlight how performance of firms, in the form of profits and efficiency, are influenced by variations in market power (MC/PC vs Monopoly/oligopoly?). Students are then expected to highlight factors besides its market power that could influence its performance.  Suggested Outline  Introduction ‐ Performance of firms in a market is often measured by looking at several different criteria – type 

of profits in the long run, allocative efficiency, productive efficiency and dynamic efficiency. For different groups of agents in society, performance of a firm would focus on different areas.   

Body Thesis: The performance of firms in a market is determined by market power   Performance 1. Profits: ‐ A  competitive market,  for  example  perfect  competition,  has  a  large  number  of  buyers  and 

sellers, homogeneity of product and no barriers of entry.  ‐ Flatter demand curve facing firm due to low barriers to entry suggests no/limited market power 

for firms to raise prices (price‐taker) to earn supernormal profits in long run.  ‐ Would  only  make  normal  profits  in  the  long  run  as  the  supernormal  profits  would  entice 

competitors who can easily enter and exit the  industry. Due to  low barriers to entry firms  lack market power in influencing prices. As a price taker, a single firm has no power to raise prices to earn more profits. 

‐  [Diagram showing PC OR Monopolistic Comp Eg. earning normal profits in LR] 

 ‐ In contrast, firms in an oligopolistic industry and a monopoly are likely to earn supernormal 

profits in the long run due to the high level of barriers (steep DD curve facing firm) new firms are 

prevented from entering thus conferring market power to existing firm/s 

‐ This allows firms to restrict output to raise prices, the market power to do so allows them the 

possibility to earn supernormal profits in the long run.  

[Diagram showing Monopoly OR Oligopoly eg. earning supernormal profits in LR]  

‐ Extent of market power is determined by barriers to entry. The greater the degree of BTEs the more market power a firm has in setting higher prices thereby earning greater supernormal profits in the long run  

 2. Productive efficiency ‐ Productive efficiency  is  achieved when  the  firms  in  an economy  are producing  the maximum 

output  for  the  given  amount  of  inputs,  or  producing  a  given  output  with  the  least  cost combination of inputs. 

‐ All firms may be productively efficient. ‐ However, under perfect competition, where firms have a lack of market power; in the long run, 

competition  forces  perfectly  competitive  firms  to produce  at  the point of minimum  average total cost of production and to charge that price which is just consistent with these costs.  

Page 32: 2016 Economics JC2 H2 (9732/01) NYJC

‐ Firms with greater market power (monopoly) may tend to be more X‐inefficient. Given greater market power may  imply  limited  competition  therefore  complacency  in minimising  costs may set in. 

 3. Dynamic Efficiency (Variety of products and Innovation) ‐ Absence of barriers to entry  implies that there are no super‐normal profits to make  in the  long 

run and this means that firms do not have the resources to engage  in  innovation. So Firms  in a perfectly competitive market lack the ability to innovate. No product or process innovation. 

‐ Oligopolies and monopolies can maintain supernormal profits in the long run, this is due to their ability to raise prices and therefore earn supernormal profits. As a result, more able to carry out research and development to create new products and increase the variety of products available to consumers. Therefore they can improve product type and improve production processes. 

‐ Firms under monopolistic competition have  the highest  incentive to differentiate because  the monopolistically competitive firm competes directly with many other small sellers in the same market.  

‐ Evaluate  extent of DE ‐ Depending on whether the firms are a collusive or competitive oligopoly, the incentive to innovate 

is different. Under a collusive oligopoly, firms may agree to limit advertising expenditure or even product development, because  retaliatory advertising and  extensive  spending on R&D by  each and every firm may be self‐cancelling and reduce total industry profits.  

‐ On  the  other  hand,  competitive  oligopolies may  have  an  incentive  to  create  new  products  to differentiate their product  in order to raise the demand  for their product and capture a greater market share. Firms that do not innovate or try and create some sense of brand loyalty may see themselves losing market share to their rivals.  

 Allocative Efficiency ‐ Explain Allocative Efficiency ‐ Allocative efficiency is only present under perfect competition due to the presence of many 

competitors all selling the same product. A PC firm faces a perfectly price elastic demand curve, 

where the marginal revenue of selling a product is exactly equal to the marginal cost for selling 

an additional unit. Assuming a profit maximising firm, the PC firm will charge consumers at 

P=MC.  

‐ For a firm that has very little market power, in the face of significant competition (PC) and the 

inability to exert any influence over increasing its price it will be more likely for the firm to set 

prices closer to its MC rendering it closer to allocative efficiency. 

‐ Hence, it is possible to assert that the extent of how allocatively efficient a firm might be, as 

reflected by the slope of its demand curve, would be that the less market power it has the more 

allocatively efficient it will be. 

 Anti‐thesis:  The performance of  firms  in a market  is not determined by market power – Other factors   Regulation: ‐ Governments often play a  regulatory role  in protecting the welfare of consumers  in society.  In 

cases where  an  industry  is  considered  crucial  such  as  utilities  but  is  a  natural monopoly,  the government  is  likely to regulate the market to force these firms to produce a  larger output at a lower  price  than  they  otherwise would  have  if  unregulated.  (AC  pricing? MC  pricing?  Affects supernormal profits and efficiency) 

‐ A firm’s performance could also be affected if a firm is nationalised. Nationalisation may give rise to  inefficiency  due  to  government  bureaucracy  and  corruption.  In  the  absence  of  the  profit 

Page 33: 2016 Economics JC2 H2 (9732/01) NYJC

motive, governments may not  run nationalized  companies as efficiently as a private company. Hence, firms could become X‐inefficient as they become lax with cost controls.  

‐ A  firm’s  performance  in  this  case  would  be  determined  by  government  intervention  in  the market.  

 Alternative objectives of firms: ‐ The  traditional  theory of  firms assumes  that  the main objective of  firms  is  to profit maximise. 

However,  in  reality,  firms  may  choose  alternative  goals  other  than  profit  maximisation particularly in the short run.   

Contestability: ‐ The theory of contestable markets argues that what is crucial in determining a firm’s behaviour 

and hence performance  is not the actual amount of competition but whether there  is the real threat of competition. 

 Prevailing economic conditions, nature of goods sold, etc: ‐ E.g. during a recession, demand  for  luxury goods sold will  fall while demand  for  inferior goods 

will rise. A firm which has high market power (e.g. oligopolistic firm) but selling luxury goods may suffer from losses while a firm which has little market power (e.g. monopolistic competitive firm) but selling  inferior goods may earn profits. This shows that market power is not the factor that determines the performance of firms.  

Conclusion/Synthesis: ‐ In theory, the performance of a firm in a market is influenced by market power. ‐ However,  there  are  several  other  factors  in  reality  that  affect  the  performance  of  firms. 

Governments can choose to regulate monopolies to ensure that the welfare of consumers are protected,  managers  can  choose  to  pursue  alternative  objectives  to  maximise  their  own benefits  rather  than  to  profit maximise  and  the  threat  of  competition  could  alter  a  firm’s behaviour and hence performance.  

‐ The view that the performance of firms in a market is determined by market power is likely to hold  true under  conditions  that  firms do not deviate  from  the  traditional objective of profit maximisation and in the absence of government intervention.    

Level  Descriptors 

L1 (1 – 2)   (3 – 5) 

Largely irrelevant answer with a few valid points.  

Some knowledge of how market power in different markets affects firms’ performance but there may be conceptual errors or lack of development.  

Little or no attempt to compare across market structures.  

No  consideration  of  how  market  power  is  not  the  only  factor  that  determines  the performance of firms in a market. 

L2  (6 – 8) 

Accurate explanation of how market power in different markets affects firms’ performance with adequate development.  

Considers performance of firms  in terms of  impact of at  least 2 criteria (firms’ profitability and/or  society’s  concerns  of  efficiency  (productive  or  allocative  or  dynamic)  and/or consumer’s welfare (variety or product innovation).)  

Some  attempt  to  compare  across market  structures. Answer mentions  at  least  2 market structures. 

For 8m there must be consideration of at least 1 other factor of how market power is not the only factor that determines the performance of firms in a market.  

Page 34: 2016 Economics JC2 H2 (9732/01) NYJC

L3 (9)         (10 – 11) 

Well‐developed  explanation  of  how  market  power  in  different  markets  affects  firms’ performance.  

Considers  performance  of  firms  in  terms  impact  on  at  least  3  different  criteria  (firms’ profitability and  society’s concerns of efficiency  (productive or allocative) and  consumer’s welfare (variety or product innovation).) Including at least 1 type of efficiency. 

Good  comparison  of  how  degree  of market  power  across market  structures  affects  the performance of firms.  

Good consideration of at least 1 other factor of how competition is not the only factor that determines the performance of firms in a market.  

Excellent explanation of how market power in different markets affects firms’ performance. 

Considers performance of firms in terms impact on all 3 different criteria (firms’ profitability and  society’s  concerns  of  efficiency  (productive  or  allocative)  and  consumer’s  welfare (variety or product innovation).)  

Excellent  comparison  of  how  market  power  across  all  4  market  structures  affects  the performance of firms.   

Excellent consideration  (at  least 2  factors) of how competition  is not the only  factor that determines the performance of firms in a market. 

Evaluation 

E1 (1–2)  Mainly unexplained judgment. 

E2 (3–4)  Well‐explained judgment on whether competition is the only factor that affects the performance of firms. 

 

Page 35: 2016 Economics JC2 H2 (9732/01) NYJC

3  Shisha smoking was banned in Singapore in 2014 because smokers were at risk of 

developing  the same health problems as cigarette smokers, such as cancer and 

heart disease. 

(a)  Explain why government  intervention is advocated  in the markets for both 

merit and demerit goods.  [10] 

(b)  Discuss whether a ban is the best response in addressing the market failure 

arising from shisha smoking.  [15] 

Suggested Answers: 

(a)

Command Word: Explain why  Content: Merit good, demerit good, sources of market failure (positive externality, negative 

externality, imperfect info) Context: Examples based on merit and demerit good 

Synopsis: To give reasons why government intervention is advocated in the markets for both merit and demerit goods making reference to externalities and imperfect information in explanation. 

INTRODUCTION 

Define merit good and demerit goods

Government intervention is advocated because…o Market fails (define market failure)o Issues: underconsumption/production, overconsumption/productiono Inefficient allocation of resources  deadweight loss

BODY 

Merit Good  Vaccination Positive Externality 

Define positive externality

In deciding how much medical products  to consume  consumers only consider  theirprivate benefits  protection from diseases when they receive the vaccination, but donot  take  into account  the positive  impact on  the health of  close  family members andfriends around  them  (third party) as  they are protected  from contracting diseases  tooand therefore can save on medical expenses (external benefit)

Existence  of  positive  externality    divergence  between  private  benefit  and  socialbenefit

Figure 1  assuming that there are no external costs, the level of vaccination taken is atQp where MPC=MPB where consumers satisfy their self‐interest

Page 36: 2016 Economics JC2 H2 (9732/01) NYJC

2  

           

However,  the  socially  optimal  level  of  consumption  of  vaccination  is  at  Qs  where MSC=MSB where society is better off.  

Qp  < Qs   under‐consumption  of  vaccination which  leads  to welfare  loss  to  society shown by the shaded area A  

Government intervention is advocated due to… o Under‐consumption  of  vaccination  that  generates  positive  externality  which 

therefore should be consumed more o Free market fails to allocate resources in a way that maximizes society’s welfare   deadweight  loss   government  intervening  to  increase  consumption  will reduce this deadweight loss 

 Demerit Good  Shisha tobacco products Negative Externality 

Define negative externality 

In deciding how much shisha to smoke  smokers only consider their private costs  price of shisha, but do not  take  into account  the negative  impact on the health of  the passive smokers around them  external cost accrued to the second‐hand smokers due to the action of shisha smokers smoking   includes the healthcare cost (external cost) that  second‐hand  smokers  (third  party)  have  to  incur  from  potential  health  risks  of having to inhale smoke from shisha smokers smoking  

Existence of negative externality  in shisha smoking  divergence between private cost and social cost  

Figure 2  assuming that there are no external benefits, the level of shisha smoked is at Qp where MPC=MPB where smokers satisfy their self‐interest            

 

Page 37: 2016 Economics JC2 H2 (9732/01) NYJC

3  

However,  the  socially  optimal  level  of  consumption  of  cigarettes  is  at  Qs  where MSC=MSB where society is better off.  

Qp > Qs  over‐smoking  shisha which  leads  to welfare  loss  to  society  shown by  the shaded area A  

Government intervention is advocated due to… o Over‐consumption of shisha tobacco products that generate negative externality 

which therefore should be consumed less o Free market fails to allocate resources in a way that maximizes society’s welfare   deadweight  loss    government  intervening  to  reduce  consumption  will reduce this deadweight loss 

 Imperfect information 

Define imperfect information 

Imperfect information due to the ignorance of the actual private cost  under‐estimation of private costs 

o Actual private cost  healthcare cost associated with the increased probability of contracting lung cancer and heart disease due to smoking shisha as well as additional healthcare cost from contracting influenza or tuberculosis due to unhygienic sharing of mouthpieces attached to shisha smoking equipment  

o Perceived private cost  price of smoking shisha  

          

Figure 3  MPCactual > MPCperceived  over‐smoking of shisha by QactualQperceived  

Government intervention is advocated due to… o Over‐consumption  of  shisha  tobacco  products  that  brings  about  greater 

healthcare costs and risks in the future and therefore should be consumed less o Free market fails to allocate resources in a way that maximizes society’s welfare   deadweight  loss    government  intervening  to  reduce  consumption  will reduce this deadweight loss 

 CONCLUSION 

Government intervention is advocated in the markets for both merit and demerit goods due to the issues of under and over‐consumption as well as deadweight loss. 

Policies are therefore required to address the market failure arising from the consumption of merit and demerit goods.    

Page 38: 2016 Economics JC2 H2 (9732/01) NYJC

4  

Level  Descriptor  Marks

L3  A clear and developed explanation of how the market fails for both merit and demerit goods (externalities and imperfect information) with links to why government intervention is advocated. 

7‐10 

L2  Undeveloped explanation of market failure due to both merit and demerit goods. Answer  does  not  explain  on  imperfect  information  but  has  examples  of negative and positive externalities. (max 6 marks) 

 5‐6 

L1  Generally descriptive in nature, limited reference to economic framework.  1‐4

    

Page 39: 2016 Economics JC2 H2 (9732/01) NYJC

5  

(b)  Command Word: Discuss whether (2‐sided) Content: Ban, alternative policies, evaluate policies Context: Shisha smoking  Synopsis: To look at the effectiveness of ban in addressing the market failures associated with shisha smoking (negative externality and imperfect information) and after which providing alternative policies. Assessment on the policies given as well as deducing if ban is the best are required.   INTRODUCTION 

Define market failure 

Source of market failure o Negative externality in smoking shisha o Imperfect information (difference between perceived cost and actual cost of 

shisha smoking) 

Policies  Ban, Tax, Education  BODY  THESIS: Ban is the best response in addressing the market failure arising from shisha smoking  

Ban – legal prohibition on the smoking of shisha  output consumed = 0  Qs = 0 (Figure 4)  

             

Ban is the best way  no external cost will incur on third party since smoking shisha generates extremely high external cost  addresses  market failure due to over‐smoking of shisha by reducing consumption to 0=Qs 

     

Page 40: 2016 Economics JC2 H2 (9732/01) NYJC

6  

ANTI‐THESIS: Ban is not the best response in addressing the market failure arising from shisha smoking as there are limitations to banning and therefore, alternative policies are required to address the market failure  Limitation of Banning Shisha Smoking 

Government failure  banning shisha results in greater deadweight loss compared to before  

           

Figure 5  Shisha smoking is banned at Qp’  banning does not reach socially optimal output Qs  deadweight loss after ban is imposed, Area B, is larger than not intervening at all, Area A  banning is ineffective in addressing the market failure due to shisha smoking since deadweight loss still exist 

Retailers selling shisha tobacco  loss in revenue   Alternative Policy 1: Tax 

Tax imposed  increases price of shisha  increases cost of smoking shisha  increases MPC  

 

          

Figure 6  assuming amount of tax = MEC  shisha smoking reduced from Qp to Qs  deadweight loss is eliminated  efficient allocation of resources  market failure from shisha smoking addressed 

Taxation is a better way than banning  o Forces smokes to be responsible for the external costs accrued to second‐hand 

smokers  smokers will then have to reduce the quantity of shisha smoked o Flexibility  tax according to MEC 

Page 41: 2016 Economics JC2 H2 (9732/01) NYJC

7  

Limitations of taxation o Government failure  underestimation of MEC  under‐taxing  shisha 

smoking not reduced to socially optimal output Qs  deadweight loss still exist  o PED < 1  due to addiction  less than proportionate increase in quantity 

demanded when price of shisha smoking rises due to taxation  negligible effect in reducing shisha smoking 

o Taxation is not able to address market failure due to imperfect information  require alternative policy 

 Alternative Policy 2: Education 

Educational campaigns by Health Promotion Board in Singapore  keep shisha smokers more informed of the actual private costs of smoking  e.g. Singtel mobile phone subscribers who come within 1km of shisha hot spot at  Kampong Glam received multimedia message in the form of an 18‐second video informing them about the actual costs of smoking shisha which is not just the health risks from smoking tobacco products but also the possibility of contracting other diseases such as tuberculosis or other viruses left behind by the previous shisha smokers via the mouthpiece as not all shisha retailers wash their equipment regularly  informing shisha smokers of such facts  increases their perceived private cost of shisha smoking 

              

Figure 7  educating shisha smokers increase their perceived costs of smoking from MPCperceived to MPCperceived+education  shisha smoking up till quantity Qperceived+education where this is the socially optimal level of output 

Limitation of educational campaigns   o Whether or not quantity of shisha smoked is reduced because of the 

campaigns are voluntary  might only have minimum impact on the level of consumption 

o Opportunity costs involved in implementing educational campaigns  resources could be better used to finance other programmes that could have a more certain impact 

   

Page 42: 2016 Economics JC2 H2 (9732/01) NYJC

8  

CONCLUSION 

Shisha was banned due to large health risks associated with the smoker as well as external costs accrued to passive smokers. Although it does reduce smokers’ smoking of shisha in Singapore, shisha smokers could still seek alternative methods to satisfy their addiction  smoking cigarettes or smoking shisha overseas  defeats the intention of banning and health risks to smokers sustains 

Although banning does not address the root cause of market failure due to imperfect information and therefore other policies are required, it allows for quantity of shisha smoking to be at socially optimal level (Qs) = 0 

Banning is the best response in addressing the market failure due to shisha smoking but there must be monitoring and enforcement in place to ensure that retailers oblige to the policy of banning. Banning could also be about banning shisha smoking for those who are of a certain age and banning the activity of shisha smoking in some places instead of a complete ban. 

  

Level  Descriptor  Marks

L3  3 (Ban & 2 others) well‐explained policies with limitations to address the market  failure  (negative  externality  and  imperfect  information)  in context. 

9 ‐ 11  

L2  2  (Ban & 1 other) well‐explained policies with  limitations.  (attempts  to answer according to context)  2  (Ban  &  1  other)  well‐explained  policies  without  limitations.  (max  6 marks) 

7 – 8   6  

L1  For  an  answer  that  shows  some  knowledge  on  policies  taken  by  the government to address market failure. 

1 – 5 

E2  For an evaluative assessment based on economic analysis. 3‐4

E1  For an unexplained assessment or one that is not supported by analysis.   1‐2

       

Page 43: 2016 Economics JC2 H2 (9732/01) NYJC

4. Amid the slowest GDP growth in six years and the flattest employment growth inmore than a decade, Finance Minister Heng Swee Keat announced a Budget with astrong focus on economic restructuring such as tightening the foreign labourmarket to raise productivity.

Source: channelnewsasia.com/news/singapore/budget-2016

(a) Explain the key determinants of sustained economic growth. [10] (b) How far do you agree that the Singapore government should focus on raising

productivity to achieve sustained economic growth? [15] a) Intro:

- Define ‘economic growth’ (actual and potential). - Outline the 2 primary sources of growth, higher AD and higher AS. - Address the ‘sustained’ nature of the growth in question.

Body 1 (AD) - Show how an increase in AD will lead to an increase in RNY (diag). - Explain the ‘key’ factors that can increase AD. - Explain what influences the levels of C, I, G and (X-M) individually (using illustrative

examples of different economies). - Identify which of these factors are most important.

Body 2 (SRAS) - Show how an increase in SRAS (downward shift) will increase RNY (diag). - Explain the ‘key’ factors that can shift AS downward (CoP). - Explain the key components of CoP (i.e. wages, interest, rents). - Comment on the viability of this as a source of ‘sustained’ growth.

Body 3 (LRAS) - Show how an increase in LRAS (rightward shift) will increase RNY (diag). - Explain the key factors that can shift AS rightward (quantity & quality of FoP). - Explain how an increase in the quantity & quality of land, labour and capital can

increase RNY (using illustrative examples).

Conclusion: (Summative) - Although summative in nature, the conclusion here must be agglomerative in that it

sums up the key factors (AD, LRAS and LRAS. Comment on the relative importance of the different factors.

MARK SCHEME L1 Shows a cursory knowledge of the factors affecting economic growth.

Little or no use of the AD/AS framework. 1-4m

L2 - Shows good knowledge of the factors affecting economic growth. - Is able to categorise these factors into the framework:

Actual growth (AD and SRAS factors) Potential growth (LRAS factors)

- Able to recognise that to achieve sustained growth, BOTH SR and LR factors must be present.

[Students who leave out SRAS factors should not be penalized]

5-6m

L3 - Able to explain 3 key determinants involving actual and potential growth with the use of real-world examples.

- Analysis considers the context of different economies (e.g. Developing, developed, emerging etc.)

7m

8-10m

Page 44: 2016 Economics JC2 H2 (9732/01) NYJC

(b) Intro: - Outline the different ways to achieve sustained growth increasing productive

capacity (increasing the quantity & quality of FoPs) whilst maintaining AD growth. - Outline the context of the Singapore economy

Constraints: small geographical size, shrinking size of the domestic work force, lacking in natural resources etc.

Condition: slow GDP growth, flat employment growth (pre-amble) - Need for policies that increase LRAS while addressing the need to boost weak AD. - Productivity affects quality of FoPs raises LRAS (and subsequently SRAS). Body Thesis: Government should focus on raising productivity to achieve sustained EG - Explain how the Singapore government raises productivity (e.g. tightening of foreign

labour market (pre-amble), use of technology etc.) - Explain how raising productivity affects the LRAS curve (diag). Anti-Thesis 1: Limitations of raising productivity to achieve sustained EG - Explain the limitations and potential difficulties of raising productivity.

Resistance of firms to use technology (when cheap alternatives are available)** Resistance of workers to new methods and re-training. Results will take time to be reaped.

Anti-Thesis 2: Use of other policies to achieve sustained EG (a) DD-management policies (highlight urgency given weak economic conditions) - Using the Singapore context, show how DD-management policies (either FP or ERP)

can increase RNY (diag). - Explain the limitations.

Singapore has a small K Implications on the government budget

(b) Other SS-side policies Infrastructure development (e.g. MRT lines, airport terminals etc.) - Use illustrative example to explain how SS-side policies can affect the AS curve (diag) - Explain the limitations and potential difficulties of SS-side policies

Adds a strain on the govt budget. Long run approach which only offer rewards in the very long term. If not properly chosen some projects may turn into ‘white elephants’.

SS-side measures to raise quantity of FoP (e.g. easing PR and labour requirements) - Use illustrative example from the Singapore context to explain how the increase foreign

labour affects the AS cure (diag). - Explain the limitations and potential difficulties of increasing foreign labour.

Worsens income inequality. Reduces the incentive to improve production methods (productivity)** Only increases productive capacity in the SR. Social problems.

(Compare the effectiveness of raising productivity with other approaches)

Criterion Productivity Population (Foreign)

Infrastructure AD mgt policies

Effectiveness Effective Reduces costs; increases productive capacity in the SR

Can be ineffective if not well chosen

Effective but insufficient.

Costs (Time frame)

Cost of education and retraining. Difficult to do LT.

Low costs to the govt. Social costs are high. -ve effects on Y-distrn

Cost of infrastructure projects can be high.

Budget implications.

Sustainability Can be sustainable if a productivity ‘culture’ is developed.

Not sustainable Not very sustainable (limit to infrastructure projects).

Page 45: 2016 Economics JC2 H2 (9732/01) NYJC

Conclusion: (Evaluative) - A comment should be made on the need for the government to manage AD in order to

achieve sustained growth. - A summary of the results from the above comparison should be made. - A judgement must be made wrt the extent to which increasing productivity should be

the government’s primary focus. (Time period, prevailing macro threats)

(Example: “Although increasing productivity would be the most desirable approach, DD management policies should also be considered in times of imminent economic slowdown.”)

MARK SCHEME L1 Shows knowledge of how productivity can affect growth. No application of

theoretical framework to support analysis.

1-5m

L2 - Able to explain how sustained economic growth can be achieved by raising productivity and one other policy (AD or LRAS) using ADAS framework.

- Able to explain the limitations of the chosen measures. [Max 7m for explanation of supply-side policy as the alternative measure.]

6-8m

L3 - Able to explain how sustained economic growth can be achieved by raising productivity and two other policies (AD and LRAS) using ADAS framework.

- Able to illustrate the policy approach and its limitations in the Singapore context. (Other economies could be used to illustrate the limitations).

- Able to compare the different policy approaches.

9-11m

E1 Makes a judgement but lacking in economic justification 1-2m

E2 Well substantiated judgement based on economic analysis and application to the Singapore context.

3-4m

 

Page 46: 2016 Economics JC2 H2 (9732/01) NYJC

a) Explain the different types of unemployment an economy can face? (10)

Introduction

Define unemployment - Refers to people who are registered as able, available and

willing to work at the going wage rate in a suitable job but who cannot find paid

unemployment despite an active search for work.

Introduce the 3 main types of unemployment – structural, cyclical and frictional

unemployment.

Body i. Structural unemployment

- Structural unemployment refers to the mismatch between the skills possessed by

the retrenched workers and those required by new industries. It is part of the

natural rate of unemployment and exists even when the economy is at full

employment as mentioned at the start of the essay.

- Results mainly from immobility of resources (e.g. geographical and occupational

immobility) when the structure of the economy changes or when there are

permanent changes in demand and supply conditions.

- Explain occupational and geographical immobility.

- Example: Singapore moved from manufacturing (especially in the areas of

chemical, electronics and engineering) in the 1990s to R&D (especially in the

areas of environmental & water technology, biomedical sciences and interactive

& digital media) in the millennium.

ii. Cyclical unemployment

- Cyclical unemployment: occurs when the economy is at the recessionary phase

of the trade cycle. It is caused by a decrease in aggregate demand (from either

internal or external problems). Internal → Recession → C & I fall → AD fall →

real output fall → demand for FOP fall → demand deficient unemployment.

External → Global downturn → X fall → AD fall → real output fall → demand for

FOP fall → demand deficient unemployment.

Page 47: 2016 Economics JC2 H2 (9732/01) NYJC

 

- Example: Singapore experienced cyclical unemployment during the global

financial crisis in 2008 which caused decrease in demand for Singapore’s

exports. When X falls, demand for labour would fall; bringing about cyclical

unemployment

iii. Frictional unemployment

- Frictional unemployment is associated with normal labour turnover and

aggravated imperfect market knowledge (due to ignorance of job opportunities). It

is part of the natural rate of unemployment.

- Example: Job hopping in Singapore especially in the finance and accounting

sector due to better remuneration, promotion, a new challenge, an overseas

posting or more flexible work arrangements.

Conclusion Singapore’s unemployment level is very much caused by internal and external factors.

Structural unemployment is the most significant problem in Singapore as it restructures in

the face of globalisation.

Marking scheme

Level Descriptors Marks 1 Answer is largely vague in definition. Only few valid points

made incidentally.

1-2

Answer shows some knowledge of unemployment but the focus predominantly on the types of unemployment rather than the causes.

3-4

2 Evidence of ability: a. Identify the appropriate types of unemployment

(structural, cyclical, frictional) b. Explain the causes to these types of unemployment

although underdeveloped explanation No or weak examples given.

5-6

3 Evidence of ability: a. Identify the appropriate types of unemployment

(structural, cyclical, frictional) b. Good explanation of the causes to these types of

unemployment Some good examples were given.

7-8

A thorough knowledge of the 3 main types of unemployment.

9-10

Page 48: 2016 Economics JC2 H2 (9732/01) NYJC

 

Demonstrate a good ability to describe and explain in a precise, logical and reasoned manner.

Good examples were given throughout the answer. Conclusion is stated.

b) Discuss the view that structural unemployment is the most significant macroeconomic problem faced by open economies. (15)

Introduction

Define open economy

- An open economy is one with no barriers to free market activity which is

characterized by the absence of tariffs, taxes, licensing requirements, subsidies,

unionization and any other regulations or practices that interfere with the natural

functioning of the free market.

State the macroeconomic problems faced by open economies.

Body Thesis: Structural unemployment is the most significant macroeconomic problem faced by open economies.

i. Effects of structural unemployment in comparison with frictional and cyclical unemployment

Workers might not obtain a job when the economy recovers as their skills are

irrelevant to the new industries or they could even be displaced when the economy is

growing due to changes in comparative advantages and globalisation.

This could go on for a longer period of time compared to the other two types of unemployment which could have detrimental effects on the individual, economy,

government and society.

Due to prolonged periods of joblessness, workers become de-skilled as their skills

become increasingly dated in a rapidly changing job market which further reduces

their chances of gaining employment in the future. This adds additional burden to the government as more unemployment benefits need to be given out.

Page 49: 2016 Economics JC2 H2 (9732/01) NYJC

 

Coupled with increased spending by the government in terms of benefit payments

and lesser tax revenue, government would be experiencing a budget deficit.

Such prolonged structural unemployment would also cause the economy to slow down as peoples’ disposable income is reduced -> decrease consumption ->

decrease AD -> decrease GPL and RNY. Unemployment problem would worsen as

less labour is demanded given that it is in derived demand.

The effects of structural unemployment would be most significant for open economies given the mobility of labour. Workers who lack the skills needed in the

industries are able to move to other economies where their skills are needed hence

leading to the problem of ‘brain drain’ -> decrease in productive capacity ->

decrease in potential growth.

In addition, prolonged periods of structural unemployment would result in the lack of confidence amongst investors. Open economies would be impacted more

significantly due to free flow of capital. Hence, I decrease -> decrease AD ->

decrease GPL and RNY.

ii. Effects of structural unemployment in comparison with other macroeconomic problems

It takes a shorter time to address the other macroeconomic problems (such as slow

economic growth, recession, inflation) compared to structural unemployment as it

takes time to train and/or re-educate workers and the success depends very much on

the capacity of workers to learn and apply their new found skills and knowledge and

prove that they could value-add to firms which are using technology in their

production processes.

Given the openness of the economy, prolonged structural unemployment could result

in decrease productive capacity -> increase GPL -> decrease price competitiveness

of Xs -> decrease in (X-M) -> decrease AD -> decrease GPL and RNY and BOP

deficit due to trade and capital deficits.

Page 50: 2016 Economics JC2 H2 (9732/01) NYJC

 

Anti-thesis: Structural unemployment is not the most significant macroeconomic problem faced by open economies.

i. Structural unemployment might not be the most serious type of unemployment

For example, during the 2008-2009 sub-prime crisis in USA, there was decrease in

consumption expenditure due to decreasing income and decrease in investment

expenditure due to declining profits and poor business sentiments. Hence, cyclical

unemployment arises.

It may affect a few industries at first but as business pessimism grows,

unemployment will spread from one industry to another. This fall in unemployment is

expected to affect all industries in the economy at the same time, leading to mass

unemployment. In comparison, structural unemployment usually only affects specific

industries thus only specific groups of workers will be affected which is less

detrimental than cyclical unemployment.

Developed small open economies such as Singapore is affected through trade with

the large open economies such as USA. On the other hand, emerging open

economies in general are affected through trade with the developed economies.

ii. Structural unemployment is not the most significant macroeconomic problem

Macroeconomic problems such as inflation, deflation, recession and unfavourable

BOP might be more significant as each of them are interlinked with each other and are more widespread in their effects for open economies.

Slow economic growth

- The more significant macroeconomic problems faced by developing open economies

such as Bangladesh would most likely be slow economic growth given widespread

poverty which affects consumption levels, lack of foreign direct investments given

investors’ lack of confidence in the infrastructure and political stability. The effects of

slow economic growth are as follows:

Decrease in real national income -> reduced output of goods and services ->

decrease demand for labour -> increase in unemployment rate -> decrease

disposable income and purchasing power -> decrease consumption ->

decrease AD and problems are worsened.

Page 51: 2016 Economics JC2 H2 (9732/01) NYJC

 

Investors’ confidence affected -> decrease investment -> decrease AD and

capital outflow.

High rates of inflation

- High rates of inflation is a more significant problem for emerging open economies

such as China and India given their large domestic market which increased

consumption, undervalued Yuan which increase demand for exports and investments

-> demand-pull inflation. The effects of high rates of inflation are as follows:

Loss of confidence in currency

Decrease in investment

- High rates of inflation causes unfavourable BOP (i.e. deterioration of trade balance

and long-term capital outflow).

Unsustainable economic growth

- Emerging open economies such as China faced a more significant macroeconomic

problem of unsustainable economic growth due to export-led growth model and rapid

industrialisation which caused pollution and waste problems.

Deflation

- Developed open economies such as EU face deflation due to decrease in demand

for goods and services by emerging economies such as China and increase in

supply of goods due to improvement in technology and falling oil prices.

- With decreasing general price levels, producers will not be incentivised to produce

hence output decreases. This leads to a decrease in demand for labour as it is in

derived demand hence increasing unemployment. As unemployment increases,

disposable income decreases leading to lower purchasing power hence consumption

decreases. This leads to decrease in aggregate demand and hence real national

income which further worsens the problem of unemployment and growth.

Conclusion Whether or not structural unemployment is the most significant macroeconomic problem

depends on the type of open economy (i.e. developed / developing / emerging, small /

large), situations in the economy (e.g. political situation, government objectives, government

policies) and preparedness of the government in managing structural unemployment.

Page 52: 2016 Economics JC2 H2 (9732/01) NYJC

 

Marking scheme

Level Descriptors Marks L3 Well developed and excellent two-sided discussion in the

context of the open economy.

9 - 11

L2 Under-developed two-sided discussion with no links to open economy. Developed one-sided answer with some links to the open economy. Under-developed two-sided discussion with some links to the open economy.

6

7

8

L1 For an answer that demonstrates limited knowledge about open economies (i.e. characteristics), structural unemployment and macroeconomic problems. Several misconceptions / errors were present.

1 – 5

E2 Judgment based on analysis. The evaluative comments are well-explained.

3-4

E1 Mainly unexplained judgment.

1-2

Page 53: 2016 Economics JC2 H2 (9732/01) NYJC

Globalisation has made the planet more equal. As communication gets cheaper and transport gets faster, developing countries have closed the gap with their rich-world counterparts. But within many developing economies, inequality has worsened.

Discuss the effects of globalisation on the standard of living in the different economies. [25]

Synopsis: This question requires candidates to recognize the effects of globalization on standard of living comprising material and non-material SOL. Answers should also recognise that different countries are affected differently with some examples to illustrate. The extent to which economies are affected by globalisation depends on the nature of these economies, the size of the economy, the current state in which they are in and the policy options available to the governments.

Introduction

Globalisation refers to the increasing integration of world economies leading to greaterinterdependence arising from freer flow of goods and services, labour, capital andtechnology and ideas. This is made possible due to the lowering of transport costs,enhanced communications and technological advancement.

There are benefits and costs arising from globalisation affecting material and non-material SOL. Material SOL refers to the amount of goods and services available forconsumption to each person in the country while non-material SOL refers to the qualityof life in terms of health, literacy, social aspects etc.

The impact of globalisation on SOL varies with different economies, depending on thenature of economy, size of the economy, the current state of development and policytools available to the government.

Body

A) Globalisation can lead to higher SOL in an economy

a) Benefits of free trade arising from globalisation leading to higher material SOL Globalisation facilitates free trade which according to the Theory of Comparative

Advantage (CA) can benefit all countries if each country specialises in producing thegoods in which they have CA.

A country enjoys CA when it is able to produce a good with a lower opportunity costin terms of other goods foregone. Difference in opportunity cost arises fromdifferences in factor endowments. For example:

o China has abundance of low-cost labour CA in labour-intensive goodssuch as shoes and clothing.

o Singapore has abundance of skilled labour and high-end capital CA inknowledge-based or capital-intensive industries such as pharmaceuticalproducts or semi-conductor chips.

Specialisation and trade leads to increased world output and allows economies toconsume outside their PPC [illustrate using the PPC diagram].

Due to the greater trade volume and hence greater quantity of goods and servicesproduced / consumed, material SOL rises.

Consumers also benefit from a wider variety of goods and services from othercountries and better product quality due to intense foreign competition, of all whichfurther raises material SOL.

Page 54: 2016 Economics JC2 H2 (9732/01) NYJC

Application: ‒ Developing economies like China with CA in labour-intensive industries will

experience greater employment gains leading to improved living standards. This lifts millions of low-income Chinese citizens out of poverty.

‒ Developed economies like the US would experience a greater rise in RNY. Capital-intensive products tend to be high value-added suggesting that the rise in net exports and hence RNY would be significantly greater for developed economies.

‒ Economies with small domestic markets such as Singapore would benefit from access to global markets leading to increased output and lower prices through EOS. This leads to price competitiveness of exports, generating more income and jobs. However open economies are more vulnerable to the contagion effects of a global recession. When incomes of trading partners fall, demand for foreign goods and services fall. This lowers the net exports of the economy which leads to falling RNY and employment. Need for policies to mitigate the effects of external shocks.

b) Globalisation facilitates economic growth & employment improving material SOL

Globalisation also brings about greater investments leading to a rise in FDI and increasing AD. Assuming the economy is below full employment, the rise in AD will lead to a multiplied increase in RNY and rise in employment. Hence, there are more goods and services available for consumption and greater accessibility to them, leading to a rise in material SOL [AD/AS diagram showing AD].

In the LR, rising investment on capital goods and technology transfer also leads to a rise in productive capacity and a rise in LRAS which increases the potential growth of the economy and hence future SOL [AD/AS diagram showing LRAS].

Application: ‒ Developing economies gain from an inflow of FDI due to their cheaper cost of

production. This aids in job creation and helps developing economies accumulate capital and technological transfer enhancing potential growth and future SOL.

‒ Developed economies benefit from outsourcing thereby lowering cost of production and enhancing competitiveness of their goods and services.

c) Globalisation enhances price stability improving material SOL

Globalisation provides more opportunities to source for cheaper and better quality imports. With the removal of trade barriers such as import tariffs through the signing of FTAs, prices of imported raw material fall, lowering cost of production.

With the free movement of labour, foreign worker inflow helps to keep labour cost low. When cost of production falls, SRAS rises leading to rising RNY and employment

and hence an improvement in material SOL [AD/AS diagram showing rise in SRAS]. Application: ‒ Small countries with limited resources such as Singapore benefit from inflow of

cheaper raw material and factors of production such as cheap foreign workers. However, these economies are also more susceptible to external shocks such as droughts in the US and Middle-East crisis leading to spikes in food and oil prices. Need for policies to mitigate the effects of external shocks.

d) Globalisation improves non-material SOL

Higher growth will generate greater tax revenue due to rising incomes ( income tax) and profitability ( corporate tax) while sales tax will also increase.

Tax revenue can be spent on infrastructure development, increased access and quality of education and healthcare etc. which improves non-material SOL.

This is especially felt in undeveloped economies especially in rural areas that lack basic infrastructure and amenities.

Page 55: 2016 Economics JC2 H2 (9732/01) NYJC

B) Globalisation may result in lower SOL a) Structural unemployment due to loss of CA leads to a fall in material SOL

With globalisation, there can be a loss of CA arising from technology and skills transfer, allowing other countries to compete more effectively. For example, China has been moving up the value chain in recent years and is increasingly producing mid to high-end electronic products such as telecommunications equipment (e.g. Xiaomi smart phones).

Higher cost countries may lose CA as firms relocate to economies with lower cost of production leading to job losses. When retrenched workers in declining industries do not have the skills needed for the expanding industries, structural unemployment will occur. Material SOL will fall due to loss of incomes.

Application: ‒ Developed economies like US are more susceptible to losing their CA to cheaper

production destinations from developing economies as they move up the value chain. This could potentially lead to structural unemployment but the impact depends on the ability of the government to pre-empt potential job losses by looking into new areas of growth and training their workforce for these jobs.

b) Globalisation worsens income inequality leading to a fall in SOL

With greater labour mobility, low-skilled workers in a country need to compete with the influx of cheaper foreign workers, depressing their wages. Conversely, higher FDI inflows raise demand for higher-skilled workers who are limited in supply, causing their wages to rise. This widens the income gap. In addition, workers in export sectors tend to benefit more from globalisation than those in the domestic sectors.

As income inequality worsens, material SOL falls especially for the lower income group. A widening income gap also generates social unrest and discontent leading to a fall in non-material SOL.

Application: ‒ Globalisation has been linked to rising income inequalities in developing countries

evidenced by the growing rural-urban divide in countries such as China, India and Brazil. A report by OECD showed that average wages paid by foreign MNCs were 40% higher than those paid by local firms pointing to the uneven benefits of globalisation especially for lower-skilled workers or poor ones in rural areas. Hence the need for redistributive policies to raise SOL of this group.

c) Increased production causes environmental damage fall in non-material SOL Increased production of goods and services for trade and rising FDI leads to greater

use of resources. This gives rise to negative externalities like pollution, global warming arising from greenhouse emissions and environmental degradation, thereby lowering non-material SOL.

Application: ‒ FDI in developing economies such as those in sub-Saharan Africa tend to depend

on natural resource use and extraction (e.g. agriculture, mineral, fuel production, coal, oil and gas etc.). This leads to depletion of natural resources limiting potential growth and future SOL of these countries. Increased FDI also leads to environmental pollution arising from excessive production especially in developing countries where environmental standards and policies tend to be more lenient.

Page 56: 2016 Economics JC2 H2 (9732/01) NYJC

Conclusion and Evaluation The benefits of globalisation arising from increased trade generally outweigh the costs,

bringing about an overall improvement in SOL to world economies. However, the benefits are uneven and gains from globalization tend to vary with different

economies. Even for similar economies, the outcome tends to vary depending on factors such as the state of economy, factor endowments, government policies etc.

o Economies with small domestic markets and limited resources gain substantially from globalisation. Export-led growth driven by access to world markets and imported raw materials would significantly raise material SOL of the economy. However, such economies are susceptible to external shocks.

o Developing economies also benefit greatly from globalisation which drives their development needs through technological and skills transfers, and provides jobs through inward FDI raising material SOL. However, this comes at the expense of environmental concerns which lower non-material SOL.

o Developed economies could experience improved SOL if the economy is able to meet the changing demands and dynamism of globalisation in terms of dealing with structural rigidities leading to job losses.

The overall impact of globalisation on SOL depends on the ability of the governments to implement policies that would maximise the benefits and minimise the costs of globalisations on their economies.

Mark Scheme: (Consider the approach that uses the different kinds of economies or one uses the SoL approach)

Knowledge, Application, Understanding, and Analysis L3 For a well-developed answer that gives an explanation of both positive

and negative effects of globalisation on SOL. Analysis is supported with real life examples in context of the different economies. For a well-developed answer that considers the effects of globalisation on SOL within the context of different economies.

18 – 21

15 – 17

L2 (Must consider the different economies to distinguish)

For an undeveloped answer that gives an explanation of both positive and negative effects of globalisation on SOL to the different kinds of economies. The use of economic framework. Some application to different economies. For a well-developed one-sided answer with some use of economic framework to support the analysis.

12 – 14

10 – 11

L1 For an answer that shows descriptive knowledge of the effects of globalisation with limited attempts to link them to SOL A smattering of valid points on globalisation.

6 – 9

1 – 5

Evaluation E2 For an evaluative judgement based on economic analysis. 3 – 4

E1 For an unexplained evaluative judgement or one that is not supported by economic analysis.

1 – 2